GRABER and WILBUR PUL

Réussis tes devoirs et examens dès maintenant avec Quizwiz!

Question 2 of 4 On physical examination, you note a mildly overweight male in no distress. His vital signs are normal. His lungs are clear to auscultation. The nasal and oropharyngeal mucosae are intact, moist, and not inflamed. The remainder of the examination is unremarkable. Chest x-ray shows flattened diaphragms but is otherwise negative. You suspect GERD, but also entertain other diagnoses. Which of the following is your next step in managing this patient's cough? A Start a proton pump inhibitor B Start an inhaled steroid C Order 24-hour esophageal pH monitoring D Obtain spirometry E Obtain a chest CT

The correct answer is "A." An empiric trial of an effective gastric acid-suppressing medication in this symptomatic patient is likely to relieve the cough if the diagnosis is accurate. The ACCP recommends starting therapy with a proton pump inhibitor rather than an H2-blocker. The usual anti-reflux measures, such as avoiding fatty foods, alcohol, and food before bedtime, should be instituted as well. Prescribers must be aware that sometimes a complete resolution of cough takes months. A 24-hour pH monitor ("C") is invasive and often not necessary if an empiric trial of gastric acid suppression resolves the problem. Starting the evaluation of chronic cough with a chest x-ray is part of the ACCP recommendations, but CT scan ("E") is not indicated with a negative chest x-ray. If the cough does not resolve with empiric therapy, spirometry should be considered.

Question 3 of 4 Chest CT further confirms a parenchymal abscess in the right upper lobe with cavitation and air within the cavity. Bronchoscopy reveals pus in the airway and extrinsic compression of the bronchi. A lavage sample is obtained, but biopsies are not taken due to the clinical impression that this is a lung abscess. What organisms are most commonly isolated in lung abscesses? A Anaerobic bacteria B Aerobic bacteria C Tuberculosis D Mixed aerobic/anaerobic bacteria

The correct answer is "A." Anaerobes are isolated most often, followed by mixed anaerobic/aerobic bacteria, followed by aerobic bacteria alone (especially staphylococci).

Question 4 of 9 Six months after you discuss her findings and prescribe inhaled beta-agonist therapy, she returns with complaints of continued wheezing and difficulty breathing. Her symptoms are brought on by cold weather and exercise and she uses her inhaler two times per week or less. She woke up two nights over the last 6 months with shortness of breath and coughing. Her albuterol still works for these symptoms, but she finds them bothersome and asks, "Why haven't I gotten over this?" How would you categorize this patient's respiratory state? A Intermittent asthma B Mild persistent asthma C Moderate persistent asthma D Severe persistent asthma E Recurrent lower respiratory tract infections

The correct answer is "A." As of 2019—and incredibly—there have been no updates to the NHLBI National Asthma Education and Prevention Program (NAEPP) 2007 guidelines. According to those guidelines (Table 3-2), your patient meets the criteria for intermittent asthma. In such patients, mild symptoms correspond to an FEV1 that is greater than 80% predicted—not to be confused with the FEV1/FVC ratio which diagnoses obstructive lung disease but is not used to categorize severity.

Question 4 of 4 The three most common causes of chronic cough (cough lasting longer than 8 weeks) are: A Postnasal drip, asthma, GERD B GERD, COPD, congenital lung disease C Lung cancer, postnasal drip, COPD D Obstructive sleep apnea, respiratory infections, asthma

The correct answer is "A." Epidemiologic studies have demonstrated that most cases of chronic cough are due to postnasal drainage (often termed "upper airway cough syndrome"), asthma, or symptomatic GERD. Most cases of chronic cough seem to have only a single cause, although some will have more than one cause. Empiric therapy should be aimed at these top three causes. Of course, infection (pertussis in particular), malignancy, and other causes of cough are important to consider—and potentially rule out—as well. The evaluation of chronic cough should proceed in a logical manner. Usually, history and physical examination will find the cause. If this is unrevealing, consider a stepwise evaluation addressing each of the chronic etiologies in Table 3-6 in order. If this still does not give you an answer, consider a methacholine challenge test to see if you can reproduce the symptoms that would lead you to a presumptive diagnosis of asthma with normal spirometry.

How long are you going to maintain this patient on anticoagulation? A 3 months B 6 months C 9 months D Lifetime

The correct answer is "A." For a PE that has a reversible cause (oral contraceptive pills in this patient with a long airplane trip), 3 months of anticoagulation is adequate. For those with a second PE, lifetime anticoagulation is warranted. For those with a cryptogenic PE or PE from an acquired or inherited thrombophilia (e.g., Factor V Leiden), recommendations are all over the place from 3 months to life. Nine months is likely adequate for a patient with a PE from an irreversible cause, although patients go back to their pre-treatment risk as soon as you stop anticoagulation. And, a first PE trumps everything else in terms of risk factors for a second PE—including all those fancy thrombophilia tests! So, finding a thrombophilia does not necessarily help your decision-making process. Note that some guidelines suggest anticoagulation for life after a first cryptogenic/unprovoked PE if benefit seems to outweigh risks. This should be decided after 3 months of anticoagulation (Chest. 2016;149(2):315-352).

Question 1 of 4 You see a 38-year-old female in follow-up for a recent episode of sinusitis. The illness has been present for about 6 weeks and has not responded to 2 weeks of appropriate antibiotics. She continues to have intermittent nosebleeds, fatigue, arthralgias, low-grade fevers, and night sweats. Two new complaints have surfaced: she has a cough productive of white sputum and she occasionally expectorates quarter-sized clots of blood. She has pleuritic chest pain, but denies dyspnea, tobacco use, and cardiac or pulmonary disease. She is afebrile with a respiratory rate of 16, blood pressure 120/74 mm Hg, and pulse rate 92 bpm. Her oxygen saturation is 98% on room air. There is dried blood in the nares, but the oropharynx is clear. Cardiac and pulmonary examinations are unremarkable. Which initial test is most appropriate? A Chest x-ray B Sputum cytologic analysis C Bronchoscopy D Chest CT E CBC

The correct answer is "A." Hemoptysis is alarming to the patient and the physician—we hope. A stepwise approach is warranted with chest x-ray as the first step. Sputum for cytology might help if the suspicion for lung cancer was substantial, but the yield is likely to be low here. She may eventually require bronchoscopy if suggested by initial studies. Chest CT is likely to be part of the evaluation, but a chest x-ray should be performed first. Obtaining blood for a CBC is also important, although likely to be normal in the setting of minor hemoptysis.

Question 4 of 4 You follow the patient during her hospitalization. The next day she is more alert and is able to follow commands. Her ventilator requirements have decreased. You consider extubation. All of the following parameters predict a poor outcome for attempted weaning from ventilation EXCEPT: A Minute ventilation supplied by ventilator is <10 L/min B PaO2 <55 mm Hg while on FiO2 >35% C Rapid shallow breathing index (RSBI) of 140 D Physical examination findings of increased respiratory effort

The correct answer is "A." Preparing to withdraw a patient from mechanical ventilation—typically called weaning or liberation—relies considerably on judgment, but a few objective parameters can be helpful. In general, the patient to be liberated must be awake, alert, and cooperative. She should have reasonably good oxygenation on a lower FiO2, have PEEP <8 cm H2O, and be able to generate adequate inspiratory pressures. Minute ventilation from the ventilator of less than 10 L/min is associated with greater success with weaning. Poor prognostic indicators include a minute ventilation from the ventilator >10 L/min, PaO2 <55 with FiO2 >35%, and RSBI >105 (see Helpful Tip). Patients with poor cardiopulmonary reserve or who have significant underlying disease may also have difficulty weaning. Allow patients a period of breathing on their own (e.g., a T-piece) before extubating. This way, if the patient fails, you can simply hook her back up to the ventilator.

Question 5 of 6 Your patient returns from the surgeon much relieved. Fine-needle biopsy proved the SPN to be a hamartoma. Now your patient wants to quit smoking for good, and he thinks that he will need some assistance. You recommend nicotine replacement products and bupropion, but your patient claims to have had an allergic reaction to bupropion. Fortunately, you know of an effective alternative. To assist with tobacco cessation, you prescribe which of the following? A Varenicline B Fluoxetine C Olanzapine D Metoprolol E Clonidine

The correct answer is "A." Randomized trials have demonstrated the effectiveness of the nicotine partial agonist, varenicline (Chantix). This FDA-approved medication appears to be at least as effective as bupropion as an aid to smoking cessation. It had been thought that varenicline caused psychiatric side effects. This has been disproven in randomized, post-marketing, trials. Varenicline seems safe even for those with psychiatric disorders. However, the FDA still suggests that patients who develop depression, aggressive behavior, etc., stop varenicline and contact their provider. Fluoxetine and other selective serotonin reuptake inhibitors have not demonstrated a benefit. In schizophrenic patients, the use of atypical antipsychotic medications may aid in smoking cessation when compared with typical antipsychotics. Clonidine is sometimes used to help patients who are withdrawing from narcotics, and it may have some limited role in smoking cessation but is not very effective. You can also combine therapies: varenicline + nicotine replacement therapy (NRT) or use two types of NRT, such as gum + patch (BMC Public Health. 2015 Jul 22;15:689).

Question 1 of 9 A 20-year-old woman with no significant past medical history presents with a 2-month history of episodic shortness of breath. These symptoms began with an upper respiratory tract infection. She has fits of coughing and trouble catching her breath with exertion. She states that her breath "sounds like whistles" at times. She tried a friend's albuterol inhaler and an over-the-counter epinephrine inhaler (Primatine...yes, it is back on the market) with some improvement and wonders if she has asthma. On examination, she is breathing comfortably at 16 times per minute and her oxygen saturation is 96% on room air. Her lungs are clear to auscultation, and the remainder of her examination is unremarkable. You want to better categorize this patient's disease. Which of the following tests is most appropriate to order now? A Spirometry B Chest x-ray C Arterial blood gas (ABG) D Methacholine challenge E Chest CT

The correct answer is "A." Since this patient has symptoms of bronchospasm, spirometry will be essential in determining if there is objective evidence of obstructive lung disease. However, spirometry results are often normal in mild cases of asthma, especially when the patient is asymptomatic. Bronchoprovocation testing, with methacholine or histamine, may be useful in such cases, but should follow basic spirometry. Although chest radiography (x-ray or CT) may reveal an occult process, it is not indicated in otherwise healthy patients with symptoms of bronchospasm. Bacterial pneumonia is a potential precipitant of bronchospasm that may be diagnosed on chest x-ray, but this patient has no constitutional symptoms (like fever) associated with serious bacterial infection. Obtaining an ABG (or better yet a venous blood gas) may be helpful when a patient presents with respiratory distress but certainly not in the office setting.

Question 3 of 9 Your patient's office spirometry shows the following: Normal FVC FEV1 82% predicted FEV1/FVC 0.68 These findings are most consistent with which of the following? A Normal spirometry B Obstructive lung disease C End-stage emphysema D Interstitial fibrosis E Obesity-hypoventilation syndrome

The correct answer is "B." Always go first to the FEV1/FVC ratio. In this case, it is <0.70, which is suggestive of airway obstruction. The information provided here lacks data regarding DLCO which should be decreased in emphysema, so you could not really differentiate between chronic obstructive pulmonary disease (COPD) and asthma. But this is clearly not end-stage emphysema, so "C" is incorrect. "D" is incorrect. Interstitial fibrosis is generally marked by a restrictive pattern on spirometry and decreased TLC. Both flow rate (e.g., FEV1) and FVC are decreased in interstitial lung diseases but in proportion to each other. Thus, the FEV1/FVC is often normal or elevated. See Table 3-1 for more on interpreting spirometry results.

Question 1 of 4 A 53-year-old male is accompanied by his wife to your office and complains of a cough for 6 weeks. It is worse at night and any time he lies down. He denies sputum production, shortness of breath, chest pain, and wheezing. He takes an antacid once or twice per day to settle his stomach and notes very bad heartburn. He smoked three packs of cigarettes per day until 1 year ago, when he quit "cold turkey." He takes only hydrochlorothiazide for hypertension. He has no cardiac disorders. His wife reports that he snores at night, and she adds, "He's always hacking and clearing his throat—all night." The review of systems is negative. In order to sleep better, he has recently started having a shot (or 2 ... or 3 ...) of whiskey before going to bed. What is the most likely cause for the cough? A Gastroesophageal reflux B Lung cancer C Antihypertensive medication D Alcohol abuse E Congestive heart failure (CHF)

The correct answer is "A." This patient appears to have a chronic cough that is most likely due to gastroesophageal reflux disease (GERD). He takes antacids and exhibits throat clearing, which can be a subtle sign and is not typically identified by patients as reflux. In addition, he drinks alcohol at bedtime, further predisposing to reflux. He has a history of smoking, which does place him at increased risk for developing a bronchogenic carcinoma, but a lung mass would not be a common cause for cough. Hydrochlorothiazide is not known to cause cough (although angiotensin-converting enzyme [ACE] inhibitors are). Also, it is unlikely that symptoms would be isolated to nighttime if his cough were medication-related.

Question 5 of 5 This patient is found to have only pulmonary sarcoidosis with some mild systemic symptoms. Which of the following is the best initial choice for management? A Observation B Oral corticosteroids C Oral antibiotics D Inhaled corticosteroids E Methotrexate

The correct answer is "A." This patient has apparent pulmonary-limited disease and has minimal systemic symptoms. Nearly 50% of patients with sarcoidosis may have spontaneous resolution of their symptoms without treatment. In fact, treatment may actually prolong the disease process. If her pulmonary or systemic symptoms worsen or are causing major life problems, she should be started on oral steroids. Systemic corticosteroid therapy is the mainstay of treatment for sarcoidosis. Methotrexate and other immune-modulating drugs may be employed as well and offer a steroid-sparing effect, but these are not first-line agents. Evidence for the use of inhaled corticosteroids is lacking. Antibiotics are not effective.

Question 1 of 4 This patient's brother, who also works on a farm, notes that every time he unloads hay he has fever, cough, dyspnea, and sputum production. It tends to resolve in 2 to 5 days but reoccurs when he is re-exposed to hay. Does he wear a mask? Well, no. He would look silly and the guys would poke fun at him. Besides, none of the other workers on the farm are affected, and they are beginning to wonder if he is malingering. His examination reveals tachypnea and fine rales. There is no wheezing present. A chest radiograph shows bilateral interstitial opacities. The most likely cause of this patient's symptoms is: A Thermoactinomyces candidus (an actinomyces species) B T. sacchari C Botrytis cinerea D Cryptostroma corticale E None of the above

The correct answer is "A." This patient presents with classic symptoms of hypersensitivity pneumonitis or, in this case, "Farmer's lung." This is caused by exposure to the Actinomyces species. Acute findings include fever, chills, cough, dyspnea, and chest tightness. X-ray findings are variable and tend to be transient early on in the disease. High-resolution chest computed tomography (CT) should be obtained, which commonly shows centrilobular micronodules and ground-glass opacification in a mid-to-upper zone predominance. "B," T. sacchari, is involved in hypersensitivity pneumonitis from sugarcane (so-called Bagassosis). "C," Botrytis cinerea, is involved in hypersensitivity pneumonitis from grapes (so-called Spatlese lung). Finally, "D," Cryptostroma corticale, is involved in "Maple bark stripper's lung," another type of hypersensitivity pneumonitis—though why anyone would want to strip maple bark is beyond us.

Question 5 of 9 That pesky med student seemed to know a lot about the PERC rules and Wells criteria. But when he listed the Wells criteria, he got one wrong. The Wells criteria for PE include all of the following EXCEPT: A Estrogen use B Pulse >100 bpm C Previous history of venous thromboembolism D Clinical symptoms and signs consistent with PE E Hemoptysis

The correct answer is "A." While an important risk factor for PE, estrogen use is not included in the Wells criteria. All the others count in the Wells criteria (Table 3-4). Using a medical calculator website, such as www.mdcalc.com, is most helpful.

Question 6 of 9 Your patient goes on to develop more frequent recurrent symptoms, such that she is using her albuterol inhaler more than three times per week, although her nighttime symptoms are rare. Which medication is the most appropriate next step in treating this patient's asthma? A Inhaled triamcinolone B Inhaled salmeterol C Inhaled cromolyn sodium D Inhaled ipratropium E Oral montelukastT

The correct answer is "A." Your patient now has mild persistent asthma and should be started on an inhaled steroid. When asthma symptoms become more persistent (i.e., when they occur >2 days per week or the patient awakens from sleep >2 times per month), the inflammatory component of the disease should be addressed while simultaneously treating the bronchospastic component with short-acting beta-2 agonists. Anti-inflammatory drugs are the mainstay of chronic asthma therapy, and inhaled corticosteroids are the most efficacious with the fewest side effects. Although ipratropium, cromolyn sodium, and montelukast have a place in asthma treatment, none of these medications is a first-line agent. Ipratropium is an anti-cholinergic that works through its bronchodilatory effects, while cromolyn sodium is a mast cell stabilizer. Montelukast is a leukotriene inhibitor. The long-acting inhaled beta-2 agonists, such as salmeterol, are only recommended at Steps 3 and higher of persistent asthma control (Table 3-2, Fig. 3-1; NHLBI recommendations, 2007).

Question 3 of 9 As you attempt to rule out other potential etiologies for the patient's symptoms (e.g., pneumonia, atelectasis, and pneumothorax), you order a trusty old chest radiograph. What is the most common radiographic finding in a patient with a PE? A Pleural effusion B No acute cardiopulmonary processes C Westermark sign D Hilar/mediastinal enlargement E Hampton hump

The correct answer is "B." Admittedly, this one is a bit tricky. Approximately 75% of the chest radiographs in the setting of PE are abnormal. However, there are numerous causes for these abnormalities and none of them individually surpass the frequency of the normal chest radiographs. Specifically, the "textbook" findings of Westermark sign (loss of peripheral vascular markings) and Hampton hump (a wedge-shaped opacity due to pulmonary infarction) are infrequent, and both have a low sensitivity and low specificity. In short, all the other options can be seen as the result of a PE, but none is more frequent than a normal chest radiograph.

Question 1 of 4 A 50-year-old male who is a heavy drinker with a history of squamous cell carcinoma of the neck presents to your office complaining of abdominal pain. He has been coughing and expectorating bloody sputum and notes a low-grade fever, chills, and mild dyspnea starting about 1 week ago. He denies nausea, emesis, and chest pain. His squamous cell carcinoma was treated with external beam radiation several years ago. Examination reveals an afebrile male in mild distress. His vital signs are normal, and his lungs sound clear. The abdominal examination reveals only mild epigastric tenderness. The chest x-ray is available for your review (see Fig. 3-4). Your colleague, who is on call today, walks by and asks if you have any admissions for her. FIGURE 3-4. Question 3.6.1 patient's chest x-ray. View Full Size|Favorite Figure |Download Slide (.ppt) You consider this 50-year-old with a cough and reply: A "Yes. This gentleman will need the ICU" B "Yes. This gentleman will need a respiratory isolation room" C "No. I'm sending this gentleman home with metronidazole" D "No. I'll workup this gentleman as an outpatient"

The correct answer is "B." Because he is expectorating bloody sputum and has a cavitary lesion on chest x-ray (right upper lobe), this patient should be admitted to a respiratory isolation room until tuberculosis is ruled out. He will need further evaluation and possibly intravenous antibiotic therapy, both of which may be accomplished during his hospitalization. "A" is incorrect. There is no need to send this patient to the ICU based on his current picture. "C" is also incorrect. Metronidazole alone is not an appropriate therapy for this patient even if this is bacterial.

Question 6 of 6 As you are walking out of your patient's room, your patient says, "Hey Doc, my brother smokes too, is there a test you can do to make sure he doesn't have lung cancer?" You respond: A "Sure, I'll put in an order for him to get an outpatient chest x-ray" B "Tell him to make an appointment and we will discuss the risks/benefits of low-dose CT scan" C "Here have him expectorate in this cup, and I'll send his sputum for cytology" D "Nah, we don't recommend lung cancer screening at this time" E "Yes. We just need a Ouija board and a magic 8-ball"

The correct answer is "B." In 2013, the USPTF issued a grade B recommendation for annual lung cancer screening with low-dose CT scan in adults aged 55 to 80 years who have a 30 pack-year smoking history and currently smoke or have quit within the past 15 years. Screening should be discontinued once a person has not smoked for 15 years or develops a health problem that substantially limits life expectancy or the ability or willingness to have curative lung surgery. These recommendations are based on the National Lung Screening Trial that showed that for every 1,000 people who got screening for lung cancer, 3 fewer died of lung cancer because of screening. About 300 people need to be screened to save one life from lung cancer. Patients should be given the risks/benefits of lung cancer screening since approximately 365/1,000 patients will have a false-positive result (usually because of a benign pulmonary nodule) that leads to more CT scans (extra radiation), more invasive procedures (biopsies and surgeries), and patient stress and anxiety. Confused yet? Don't worry, the American Thoracic Society has developed a patient decision guide found at https://www.thoracic.org/patients/patient-resources/decision-aid-for-lung-cancer-screening-with-ct.php. It is important to stress that this lung cancer screening should not take the place of efforts to stop smoking! Of note, "A" and "C" are incorrect because neither plain chest radiographs nor sputum cytology have been shown to improve lung cancer mortality.

Which of the following medications, when used alone as maintenance therapy in persistent asthma, is associated with an increased risk of asthma-related mortality? A Inhaled fluticasone B Inhaled salmeterol C Oral zafirlukast D Oral prednisone

The correct answer is "B." Inhaled salmeterol, when used alone as a controller agent for asthma, has been associated with a two- to fourfold increase in the risk of death related to asthma or other respiratory conditions. Thus, the Food and Drug Administration (FDA) has mandated a "black box" warning be applied to salmeterol-containing products. It is not known whether inhaled steroid therapy is protective, but NHLBI/NAEPP guidelines recommend adding long-acting inhaled beta-agonists only after inhaled steroids are already in use.

Question 8 of 9 After a brief hospitalization, your patient recovers nicely. Prior to this incident involving aspirin, she had been free of exacerbations for about a month. In addition to a short course of oral steroids, which of the following medication regimens do you prescribe for this patient with aspirin sensitive asthma at discharge? A Inhaled triamcinolone and inhaled albuterol as a "rescue" B Inhaled triamcinolone, oral montelukast, and inhaled albuterol as a "rescue" C Oral montelukast and inhaled albuterol as a "rescue" D Inhaled albuterol as a "rescue" E Inhaled salmeterol and inhaled triamcinolone

The correct answer is "B." Leukotriene inhibitors (e.g., montelukast, zafirlukast) have demonstrated effectiveness in reducing symptoms and improving peak flow in patients with aspirin sensitive asthma. Leukotriene inhibitors should be used only in asthma patients who are already using a corticosteroid inhaler—or those who cannot tolerate inhaled corticosteroid therapy. Therefore, "C" is not an appropriate choice. "D" is incorrect because there is no anti-inflammatory drug. Although "E" offers an anti-inflammatory agent, there is no rescue inhaler, and patients with asthma must always have access to a short-acting inhaled bronchodilator.

Question 4 of 5 Which of the following is true about ACE levels in sarcoidosis? A An elevated ACE level is specific for sarcoidosis B ACE levels often correlate with disease severity in sarcoidosis C ACE inhibitors are effective in the treatment of sarcoidosis D All of the above

The correct answer is "B." One can follow ACE levels to track the progress of the disease. However, since treatment is based on symptoms, following ACE levels is not recommended. "A" is incorrect. ACE levels may be elevated in silicosis, miliary TB, and asbestosis, among others. "C" is incorrect. ACE inhibitors are not used to treat sarcoidosis.

Question 3 of 3 The most appropriate treatment for this patient with ABPA would include which of the following? A Antibiotics B Oral corticosteroids C Leukotriene receptor antagonist D Itraconazole E Inhaled ipratropium bromide

The correct answer is "B." Oral corticosteroids are the treatment of choice for ABPA. Patients are typically treated for several months with tapering doses rather than short courses of steroids. Serum IgE levels and chest x-rays are used to monitor response to treatment. Please note that "D" is incorrect, and azoles are not the mainstay therapy in ABPA. The goal of treatment is suppression of the immune system responding to the fungal antigen. There are studies that show benefit of adding an antifungal agent as a steroid-sparing agent. However, there are risks associated with concomitant use of steroids and azoles, namely, marked adrenal suppression. Hence, oral corticosteroids should be your first choice.

Question 2 of 8 When initiating supplemental oxygen by nasal cannula, you instruct the nurse to keep the patient's oxygen saturation: A Between 96% and 100% B Between 90% and 95% C Between 85% and 89% D At whatever saturation he looks most comfortable

The correct answer is "B." The primary goal of supplemental oxygen is to reduce the risk of tissue hypoxia. Maintaining oxygen saturations above 90% (or PaO2 60-65 mm) will ensure tissue oxygenation. Higher oxygen saturations may result in CO2 retention and hypercapnia, as noted earlier. Also, aiming at 100% with excessive levels of O2 supplementation takes away an important patient assessment parameter because now you cannot tell easily whether his O2 needs are going up or down. "D" is of special note. Patients with COPD who look comfortable may be developing hypercapnia and CO2 narcosis. Thus, while comfort is a goal, it may not be the best judge of clinical status in patients with COPD exacerbations. To assess CO2 levels, you will need an ABG or VBG.

Question 1 of 2 A 42-year-old male who works in a hog confinement area presents to your office complaining of cough, fever, wheeze, and dyspnea. He and some other workers were cleaning the confinement area with high-pressure hoses (which aerosolized hog waste, yum), and they all developed the same symptoms, which started between 4 and 8 hours after work. On examination, he is febrile with a respiratory rate of 28. He is able to talk in complete sentences. There are slight crackles when you auscultate the lungs. His chest x-ray is normal. The most likely diagnosis is: A Farmer's lung (hypersensitivity pneumonitis) B Organic dust toxicity syndrome C Reactive airway disease D Hydrogen sulfide poisoning E Bronchiolitis obliterans

The correct answer is "B." Organic dust toxicity syndrome (ODTS) occurs when moldy or decomposed hay and other organic material (such as hog manure) is moved. Endotoxins are aerosolized and inhaled, leading to the symptoms. The tip off here is that everyone on the job site was affected. Since hypersensitivity pneumonitis ("A") is specific to the individual, generally only one worker at the site will have symptoms. Compared with ODTS, hypersensitivity pneumonitis may present with an abnormal chest x-ray with micronodular or reticular opacities. "C" is incorrect because everyone is involved, which would be highly unusual with reactive airway disease. "D" is not correct. Hydrogen sulfide poisoning presents as a toxic pneumonitis with pulmonary edema, dyspnea, hypoxia, and loss of consciousness. Hydrogen sulfide also acts as a direct cellular toxin that binds to cytochrome oxidase system, similar to cyanide. In addition, hydrogen sulfide exposure comes from cleaning manure pits (as anyone in Iowa would know). Finally, "E," bronchiolitis obliterans, is a chronic illness rather than an acute one. Of note, there is a strong association between ODTS and subsequent development of chronic bronchitis.

Question 3 of 5 Which of the following is NOT found as a part of sarcoidosis? A Erythema nodosum B Myocardial infarction C Cardiac arrhythmias D Elevated liver enzymes E Vision loss

The correct answer is "B." Sarcoidosis does not cause myocardial infarctions. While there is cardiac involvement with sarcoidosis, the manifestations are bundle branch block, cardiac arrhythmias, and sudden death. Many organs can be affected by sarcoidosis, including the skin, eye (iritis), heart, lung, liver, nervous system—essentially anywhere granulomas form.

Question 1 of 4 A 16-year-old female comes to your office with complaints of sneezing spells, itchy watery eyes, and nasal congestion for the past 2 years. These symptoms are worse during the spring and fall and when she plays with her cat. She denies any other constitutional symptoms and has no other past medical history. She has tried over-the-counter loratadine without relief. She has lived in the same residence for the past 6 years and denies any other environmental exposures. Her examination reveals pale nasal mucosa and swollen nasal turbinates bilaterally. Her lungs and skin are clear. You are not clear if this is allergic rhinitis or nonallergic rhinopathy (previously known as vasomotor rhinitis). The most effective way to determine if this is allergic is to: A Do a methacholine challenge test B Do a Hansel stain of nasal mucus C Check overall IgE levels D Perform a nasal mucus electrophoresis to visualize allergic bands E Waft hot peppers under her nose...see what happens

The correct answer is "B." The best way to tell if this is allergic is to do a Hansel stain of the nasal mucus. This will show eosinophils if it is allergic. If this is nonallergic rhinopathy, eosinophils will be absent. If it is infectious, there will likely be a predominance of neutrophils. "A" is incorrect. A methacholine challenge test is helpful in diagnosing asthma, not allergic rhinitis. Neither "C," IgE levels, nor nasal mucus electrophoresis have any use here (and nasal mucus electrophoresis has no use anywhere that we know of... and it is disgusting). IgE is only elevated in 40% of patients with allergic rhinitis. Does anyone ever actually do a Hansel stain? Probably not, but once again, this is for the Board Examination, not real life.

Question 4 of 9 ECG and chest radiograph in hand, you turn your attention toward ordering the appropriate laboratory tests to solidify your presumptive diagnosis. You are working with a medical student who suggests a number of lab tests. You agree with most of them but shoot him down on one. Which test should you AVOID ordering? A CBC B d-dimer C PT/PTT D Basic metabolic panel (Na+, K+, Cl−, CO2−, BUN, Cr−, and glucose) E Urine pregnancy test

The correct answer is "B." The d-dimer can be a blessing for some but is the bane of existence for others. In this patient, a d-dimer is not useful. This test has great sensitivity but poor specificity. It is positive in far more conditions than PE. Used as a "rule-out" test for PE, it only applies in low-risk patients. Ms. Bellum is not a low-risk patient as suggested by the Pulmonary Embolism Rule-out Criteria (aka, "PERC"; see Helpful Tip) due to her use of exogenous estrogen. The Wells criteria for PE place her in the moderate-risk group (16.2% risk of PE). As such, even a negative d-dimer is insufficient for ruling out the diagnosis. As for the other tests, they all serve a valuable role in her evaluation. For instance, the CBC ("A") could provide evidence of anemia, while the PT/PTT ("C") may reveal a coagulopathy. Assessing her renal function ("D") may be needed for her evaluation and treatment planning, and the same can be said for verifying her gestational status ("E"). Plus, a urine pregnancy test is performed on almost every woman in an ED. It might as well be part of the triage process.

Question 1 of 5 A 35-year-old African-American female presents with dyspnea worsening over the last 2 months. She also complains of cough, generalized fatigue, and intermittent low-grade fevers. She does not smoke. Chest x-ray shows hilar adenopathy and small bilateral pleural effusions. Spirometry is consistent with a restrictive pattern. Of the following, which is the most likely diagnosis? A Granulomatosis with polyangiitis B Sarcoidosis C Bronchogenic carcinoma D Pneumonia E Microscopic polyangiitis

The correct answer is "B." The findings of hilar lymphadenopathy and a restrictive pattern on spirometry are most consistent with sarcoidosis. The chest x-ray findings do not support the diagnosis of granulomatosis with polyangiitis. Besides, it can't be "A"—we just did that case (and when would you see two GPA cases in a row on a test—let alone in your career?). "E" is incorrect. Microscopic polyangiitis is a systemic vasculitis related to GPA, presenting with similar features as GPA but without granulomatous disease. Bronchogenic carcinoma ("C") is unlikely in this relatively young non-smoker who does not demonstrate findings of carcinoma on x-ray. The clinical history is not typical of pneumonia ("D"), and chest x-ray shows no infiltrate. Tuberculosis (TB), although not an answer option, should also be considered, and the appropriate history and testing should be completed. In fact, TB and sarcoidosis often present in a similar fashion as may some fungal diseases and others.

Question 1 of 8 You are seeing a 65-year-old male in the emergency department (ED) where he presented with complaint of increasing shortness of breath. He has obvious difficulty breathing and cannot speak in full sentences. However, you are able to elicit that he has been having increasing respiratory problems over the last 3 to 4 days. He has known COPD with FEV1 of "less than one" (normal FEV1 is about 4 L for a 50-year-old male and 3 L for a 50-year-old female; for calculations based on age, etc., see www.hankconsulting.com/RefCal.html or http://www.webcitation.org/75KGIXvSb). He has been using his inhalers much more than usual but with minimal improvement. He has smoked one pack per day since age 18 (but proudly points out he quit 2 days ago) and has a past medical history of high cholesterol, obesity, and hypertension. On examination, he has a respiratory rate of 26 to 28, heart rate of 100 bpm, blood pressure of 130/90 mm Hg, and temperature of 37.7°C. His O2 saturation is 84% on room air. On auscultation, you do not appreciate much due to his body habitus, but you still manage to hear some wheezing. He has a normal cardiac examination and no lower extremity edema. What is the next best step to help this patient? A Perform emergent endotracheal intubation B Administer supplemental O2 via nasal cannula C Administer methyloprednisolone (Solu-Medrol), 1 g IV D Start antibiotics IV E Obtain a chest x-ray

The correct answer is "B." This patient is hypoxic, and your first priority should be to improve his oxygenation. There exists a theory that oxygenating patients with COPD will suppress their respiratory drive. The classical teaching (mostly incorrect) is that COPD causes a switch from carbon dioxide levels driving respiration to oxygen levels driving respiration. While this may be partly true, further study has suggested that the main reason COPD patients are at risk of worsening hypercapnia is due to loss of hypoxic pulmonary vasoconstriction and worsening ventilation-perfusion mismatch that occurs with excess oxygen delivery. Regardless of that, you need to first worry about this patient's oxygenation. He is not going to thrive with an 85% O2 saturation. "A" is incorrect. The patient is protecting his airway and you have not attempted improving his oxygenation with less invasive methods yet. Administering steroids, IV or PO, will have no immediate effect on his respiratory status. In fact, IV steroids might be worse than PO steroids (there is question of whether higher IV doses cause immunosuppression and subclinical myopathy; see Am J Respir Crit Care Med. 2014;189:1052). Antibiotics and a chest x-ray may be reasonable, but with low O2 saturations, your priority is to quickly improve your patient's oxygen status.

Question 1 of 2 A 52-year-old male smoker presents for a 3-month history of productive cough. He reports multiple episodes of pneumonia, but continues to produce copious sputum between episodes of pneumonia. Chest x-ray is unremarkable. Chest CT shows enlarged peripheral airways with thickened airway walls in the lower lobes bilaterally. Sputum culture grows several types of bacteria, including P. aeruginosa. Which of the following do you recommend as initial therapy? A Corticosteroids B Antibiotics C Chemotherapy D Supplemental oxygen E Wedge resection of the affected lung tissue

The correct answer is "B." This patient's findings are consistent with the diagnosis of bronchiectasis, a chronic inflammatory disease of the medium-sized bronchi. Appropriate initial therapy consists of prolonged courses of antibiotics, usually 2 weeks of a narrow-spectrum antibiotic followed by reassessment. Doxycycline, amoxicillin, clarithromycin, amoxicillin/clavulanate, and trimethoprim/sulfamethoxazole are often used. Respiratory quinolones demonstrate some limited use in patients with Pseudomonas. Patients should be directed to discontinue tobacco use and take inhaled bronchodilators. Resection of the affected lung tissue may be necessary but should not be the initial therapy. Supplemental oxygen therapy is used if oxygenation is poor. Chemotherapy and prolonged oral corticosteroids are not used to treat bronchiectasis.

Question 3 of 4 In this patient, which of the following ventilator management techniques will unequivocally decrease her FiO2 requirement? A Increase the respiratory rate B Increase the positive end-expiratory pressure (PEEP) C Decrease the tidal volume D Addition of inhaled nitric oxide (NO)

The correct answer is "B." Two standard techniques are usually employed to improve a patient's oxygenation: increasing FiO2 and PEEP. PEEP maintains positive pressure in the airways at the end of expiration. Its use increases lung compliance and decreases ventilation/perfusion mismatching, resulting in better oxygenation. Since FiO2 >60% over periods longer than 48 hours may result in oxygen toxicity, PEEP may be employed to reduce the need for high levels of FiO2. "A" and "C" are incorrect. Increasing respiratory rate or tidal volume will cause increases in minute ventilation, which reduces PaCO2, but has little effect on PaO2. Decreasing minute ventilation, through decreased respiratory rate or tidal volume, causes CO2 retention and increased PaCO2. "D" is incorrect. Nitric oxide has been shown to improve oxygenation in select patients with severe pulmonary hypertension and ARDS, but does not improve overall outcomes and is potentially nephrotoxic.

Question 7 of 7 Now that you have gained expertise with ultrasound of a pleural effusion, your colleague sends you a patient that has a pleural effusion on chest x-ray. Your colleague asks you whether you could "tap" the fluid for him as he does not feel comfortable with the portable ultrasound. Also, he has a tee time in 30 minutes. On ultrasound, you quickly visualize the chest and see a septated and loculated pleural effusion. What is the best next step? A Return the patient back to his doctor—the effusion is too small to access B Perform ultrasound-guided needle thoracentesis C Refer to a thoracic surgeon D Place a small-bore chest tube E Head to the golf course with your colleague. Nothing you can do here

The correct answer is "C." A loculated and septated pleural effusion can very often be seen in empyema and evacuation usually requires surgical intervention. Thoracentesis would unlikely be successful and would expose the patient to an unnecessary procedure after which he would still need to see a surgeon. Placing a chest tube blindly into a loculated pleural effusion is unsafe. That procedure should be done under visualization; most commonly, video-assisted thoracic surgery would be utilized. Intrapleural fibrinolytics may improve outcomes but should be done in consultation with a thoracic surgeon or pulmonologist. 62% of users answered correctly.

Question 2 of 4 The Hansel stain shows eosinophils. "Eureka!" you shout. Which of the following would be the most appropriate next step in managing her symptoms? A Recommend allergen-impermeable encasements for mattress and pillow B Use topical decongestant sprays C Change classes of antihistamines D Refer for allergy evaluation, including percutaneous aeroallergen skin testing E Recommend a high-efficiency particulate air filter for the home

The correct answer is "C." Although all of the above choices may provide relief for allergic symptoms, the best first step would be to try changing the class of antihistamines. Antihistamines (and NSAIDs for that matter) are grouped into classes based on chemical structure. One class may be helpful for a patient when another class does not work. The other choices are suboptimal. Without skin testing, avoidance measures may be needless, costly, and ineffective. Skin testing alone ("D") would not immediately improve her symptoms, and you would still need to do something with therapy (we hope) while waiting 3 months to see an allergist. Allergen-impermeable encasements ("A") are currently recommended for patients with dust mite allergy. Although filters ("E") are often recommended for pet allergies, the data regarding their effectiveness in reducing allergic symptoms is contradictory. Topical decongestant sprays ("B") are an inappropriate choice secondary to the addictive nature of these medications and the risk of causing rebound symptoms. Removing a pet from the bedroom may reduce—but not eliminate—allergen exposure. Further evaluation should be performed prior to recommending any such lifestyle modifications. Another reasonable option would be a trial of intranasal corticosteroid. Finally, you could also put an allergen impermeable encasement around the cat ...

Question 5 of 9 Which of the following is most appropriate for this patient given that she has intermittent asthma? A Add theophylline B Add montelukast C Continue albuterol as needed D Schedule albuterol every 4 hours E Prednisone 5 mg daily

The correct answer is "C." As already discussed, this patient appears to have intermittent asthma. She is in no respiratory distress, is oxygenating normally, and is still responding well to albuterol by her report. Although there is some debate about the role of inhaled steroids in intermittent asthma, the NAEPP and most experts do not recommend their use. Oral prednisone is certainly not indicated in this case. She should be continued on a short-acting inhaled beta-2 agonist, such as albuterol, without the addition of another medication. "D" is incorrect. Scheduled albuterol actually yields less effective symptom control than does PRN use. As to "A," a pox on those still prescribing theophylline.

Question 4 of 4 The patient is unable to change jobs or wear a respirator because it itches and he "keeps forgetting it." But he's persistent (or brave or thickheaded or unable to learn a new skill), and keeps farming. Three years later, he returns with a chronic cough, weight loss, dyspnea, fatigue, and clubbing of the fingers. Further evaluation will most likely reveal: A Bronchogenic carcinoma B Air space disease (e.g., a pneumonia-like picture) C Decreased DLCO (Diffusing capacity of the Lungs for CO [Carbon Monoxide]) D Markedly abnormal bronchoalveolar lavage (BAL) with lymphocytosis E Obstructive changes on pulmonary function testing

The correct answer is "C." Hypersensitivity pneumonitis can become chronic if exposure is not limited. In these cases, patients will generally have systemic complaints such as fatigue and possibly weight loss. Fever will be absent as may a history of prior episodes of acute hypersensitivity pneumonitis. Dyspnea and clubbing of the fingers are also generally noted, reflecting chronic pulmonary disease. Along with this finding, pulmonary fibrosis can occur and the DLCO may be decreased. "A" is incorrect. Hypersensitivity pneumonitis does not lead to lung cancer. "B" is incorrect. While acute hypersensitivity pneumonitis causes an alveolitis, chronic hypersensitivity pneumonitis causes pulmonary fibrosis with an occasional micronodular pattern. "D" is incorrect. BAL in chronic hypersensitivity pneumonitis does not contain the markedly elevated lymphocyte count that is seen with acute hypersensitivity pneumonitis. Finally, "E" is incorrect. One would see a restrictive pattern on pulmonary function testing reflecting the fibrosis and not an obstructive pattern.

Question 4 of 4 The diagnostic evaluation is in progress. Laboratory tests are pending, and a chest CT is scheduled. You have arranged for a pulmonologist to see her. When you are on call, the physician covering the ED calls you to admit her for "massive hemoptysis." When you arrive, the patient looks comfortable and has normal vital signs. She begins a fit of coughing, expectorating several ounces of bright red blood. Her systolic blood pressure falls to 80 mm Hg. Her respiratory rate is 40. Her work of breathing has increased considerably. The situation does not improve after 5 minutes of observation, and her O2 saturation is now 83% on room air. Remembering the movie Moulin Rouge (which has nothing to do with this case except for hemoptysis), what is your first action in this situation? A Arrange emergent bronchoscopy B Transfuse 2 units of blood C Perform endotracheal intubation D Provide bolus IV normal saline

The correct answer is "C." Massive hemoptysis is variably defined as 100 to 600 mL of blood expectorated per day, and it can result in hemodynamic compromise and asphyxiation. Quantification of the blood loss by the patient is usually unreliable. The main cause of mortality with hemoptysis is not hypovolemia but rather asphyxiation from blood in the lungs. As with any patient in acute respiratory distress, the airway must be controlled first. The best choice here is to perform intubation. Since this patient is known to have a potential source for bleeding in the right lung, intubation of the left mainstem bronchus may protect the left lung from the blood. Also, placing this patient on her right side (so that the bleeding source is dependent) may protect the left lung. If available, emergent bronchoscopy may allow identification of the bleeding site and selective lung intubation. However, bronchoscopy is not well suited for stopping the hemorrhage. The most that a bronchoscopist can do is place an endobronchial blocker and seal off the bleeding lobe. Interventional radiology should usually be the first treatment once the bleeding site has been localized. Emergent surgery is indicated if the bleeding remains brisk and not responsive to other interventions. Fluid resuscitation is important. However, before any of these other measures is undertaken, the airway must be protected. The patient stabilizes in the ICU. You plan to start treatment for her granulomatosis with polyangiitis. She does better and is discharged in 2 days.

Question 4 of 4 Gram stain of sputum demonstrates Gram-positive cocci and Gram-negative rods. Cultures are pending. Tuberculin skin test is negative. What is the most appropriate therapy for this patient? A Refer for surgical drainage B Oral levofloxacin C Intravenous clindamycin D Intravenous metronidazole E Intravenous ceftriaxone

The correct answer is "C." Most lung abscesses are polymicrobial, but the most important aspect in treatment appears to be the use of an antibiotic active against anaerobes. Intravenous clindamycin is the usual choice for lung abscess due to its coverage of anaerobes and Streptococcus pneumoniae. Metronidazole is less effective, failing in up to 50% of cases of putrid lung abscess. A beta-lactam with beta-lactamase inhibitor (e.g., piperacillin/tazobactam) is another good choice. Ceftriaxone and levofloxacin offer poor coverage of anaerobes. Surgical drainage of lung abscesses is needed in only 5% to 10% of cases. Most resolve with just antibiotics.

Question 2 of 4 The correct treatment for this patient with Farmer's lung includes: A Antibiotics B Inhaled steroids C Oral steroids D Leukotriene inhibitors E Bronchoalveolar lavage (BAL)

The correct answer is "C." Oral steroids are effective in the treatment of hypersensitivity pneumonitis. Neither antibiotics ("A") nor inhaled steroids ("B") are of any benefit. "E," bronchoalveolar lavage, is not a treatment. However, it can be used as a diagnostic tool. One would expect to see lymphocytes on BAL.

Question 2 of 9 If this patient has mild asthma, which of the following pulmonary function test results would you expect to find? A Forced vital capacity (FVC) 50% of predicted B Forced expiratory volume in 1 second (FEV1) 100% of predicted C FEV1/FVC ratio <0.7 D Total lung capacity (TLC) 50% of predicted E FEV1/TLC <0.7

The correct answer is "C." Patients with asthma will have a decreased FEV1. The FVC may fall as well, but FEV1 falls first and to a greater degree as the lung becomes obstructed. The ratio of FEV1/FVC is very sensitive to airflow limitations, and FEV1/FVC <0.7 (just the raw ratio of the two numbers) is generally considered diagnostic of obstructive airway disease. The rest are incorrect. TLC is not measured by spirometry (which is why "D" and "E" are incorrect); but if it were, TLC may be increased in patients with obstructive airway disease due to air trapping.

Question 2 of 2 Appropriate treatment for this patient includes: A Antibiotics B Intubation and mechanical ventilation C Supportive care D A and B E A and C

The correct answer is "C." Supportive care is the usual treatment of ODTS. Antibiotics are not needed because the syndrome is mediated by endotoxins rather than direct infection. "B" is incorrect because this patient is not in significant respiratory distress.

Question 2 of 4 Your patient is on assist-control mode of ventilation. A nasogastric tube, 2 IVs, and a bladder catheter are in place. She is given IV N-acetylcysteine. Her blood pressure improves to 112/67 mm Hg, and her oxygen saturation is 99%. Her chest x-ray shows an endotracheal tube terminating 3 cm above the carina and no infiltrates. Thirty minutes after you intubated her, with the ventilator rate at 14 breaths/min, FiO2 100%, and tidal volume at 400 mL, you obtain another ABG: pH 7.35, PaCO2 45 mm Hg, PaO2 130 mm Hg. She takes 6 to 8 spontaneous, assisted breaths, while the ventilator provides the remaining breaths. She appears to be perfusing her periphery well. Your next action is to: A Decrease the tidal volume to allow for permissive hypercapnia B Increase the tidal volume to achieve a pH of 7.45 to 7.50 C Reduce FiO2 while maintaining oxygen saturations at or above 90% D Change to pressure support ventilation

The correct answer is "C." Your patient is perfusing well, and her PaO2 and measured oxygen saturation are much improved. You should now decrease the FiO2, with the goal being to achieve an FiO2 of less than 60% while maintaining adequate perfusion and oxygen saturation. An FiO2 of 100% is somewhat toxic and can lead to airway injury. "A" is incorrect. However, permissive hypercapnia (allowing CO2 to rise to >80 mm Hg as long as the patient tolerates it) may be useful in ventilated patients with COPD, ARDS, or asthma; it allows you to reduce tidal volumes and pressures without doing harm. But remember that you still need to maintain oxygenation. "B" is incorrect. Your patient is doing reasonably well with her slightly acidotic pH, which has corrected very quickly. It may be inadvisable to attempt to increase her pH beyond 7.40, as she may develop respiratory alkalosis that can then lead to cardiac arrhythmias. Because of her low respiratory rate, she should remain on some type of assisted volume-cycled ventilation. "Pressure support ventilation," as its name implies, only augments patient-triggered breaths with increased airway pressure. This pressure is in addition to the background positive end-expiratory pressure (PEEP); PEEP is used to prevent alveolar collapse and subsequent lung injury.

The CBC, coagulation studies, and basic metabolic panel all return within normal limits. In addition, Ms. Bellum is not pregnant. Thus, you wish to (finally) solidify that diagnosis you have suspected for quite some time. Since you do not put her in the low-risk category by your clinical judgment, what diagnostic study should you order? A VQ (ventilation-perfusion) scan B CT scan of the chest without contrast C CT scan of the chest with contrast D Pulmonary angiogram E Compressive Dopplers of the lower extremities

The correct answer is "C." The American College of Radiology (ACR) lists the CT scan of the chest with contrast (i.e., CT angiography or CTA) as the modality of choice in stable patients with a suspected PE. Its benefits: it is noninvasive, cheaper than pulmonary angiography, and far more available than VQ scans. It should be noted that pulmonary angiography still remains the "gold standard" for diagnosing pulmonary emboli, but that is more of an academic point. As for VQ scans, they are not available in many locales and often return nondiagnostic. However, they can be used in a patient with a normal chest x-ray. A V/Q scan is likely to be non-diagnostic in those with an abnormal chest x-ray (e.g., COPD patients). A chest CT without contrast ("B") will not enhance the pulmonary arteries, making the diagnosis of a PE far more difficult, if not impossible. Since most pulmonary emboli are believed to arise from the lower extremity venous system, "E," Dopplers of the legs, could be considered if the patient was a poor candidate for both CTA and VQ scan (e.g., COPD and Stage 4 chronic kidney disease). But this approach is obviously not diagnostic of PE; it would just help you determine if the patient has an active thrombosis, and you would manage the same whether she has a PE or DVT or both.

Question 1 of 6 A 60-year-old male presents to the ED for a cough. His symptoms began with a cold 2 weeks ago, and the other symptoms have improved, but the cough has persisted. He has mild production of white sputum with no hemoptysis. The patient denies fevers, night sweats, chills, and weight loss. He's had no chest pain or dyspnea. He smokes one pack of cigarettes per day, works in construction, and does not have a regular doctor. In fact, with some pride, he says, "I haven't seen a doctor in over 30 years." On physical examination, you find a fit-appearing male in no acute distress. His vital signs are normal. His lung sounds are diminished bilaterally, but the remainder of the examination is unremarkable. While breathing ambient air, the patient's oxygen saturation is 94%. You obtain a chest x-ray, which is shown in Fig. 3-6. FIGURE 3-6. Question 3.11.1 patient's chest x-ray. View Full Size|Favorite Figure |Download Slide (.ppt) Your next step is to: A Prescribe a 5-day course of azithromycin B Refer the patient to a pulmonologist C Order a high-resolution CT scan of the chest D Have the patient return to you in 3 months to repeat a chest x-ray E Reassure the patient and have him return as needed

The correct answer is "C." The chest x-ray in Fig. 3-6 shows a single nodule in the right lower lobe. The nodule is round, less dense than bone, and appears to be >1 cm in diameter. These are sometimes called "coin lesions." There are no other abnormalities. The most appropriate next step in the evaluation is to order a high-resolution CT scan of the chest. Treatment with azithromycin is inappropriate in this setting, as this patient has no signs of pulmonary infection on examination or chest x-ray. Referral to a pulmonologist is premature without first investigating the nodule by CT scan. Delaying further imaging and evaluation is also inappropriate since 15% to 75% of solitary pulmonary nodules (SPNs) ≥8 mm are ultimately diagnosed as cancer.

Question 4 of 6 Initial blood cultures grow S. pneumoniae. Sputum Gram stain and culture are negative. The patient initially does well and defervesces after 2 days of IV antibiotics. However, on day 3, he again spikes a fever. He looks moderately ill. Your examination reveals increased dullness to percussion on the left. There is no jugular venous distention (JVD) or peripheral edema. The radiograph is shown in Fig. 3-7. FIGURE 3-7. Question 3.12.4 patient's chest x-ray. View Full Size|Favorite Figure |Download Slide (.ppt) The most likely diagnosis at this point is: A Anaerobic abscess B Development of resistant S. pneumoniae C Parapneumonic effusion D Transudate secondary to heart failure E Drug-induced transudate

The correct answer is "C." The most likely problem in this patient is a parapneumonic effusion. "A," an anaerobic abscess, is unlikely given that there are no air/fluid levels and the fact that the fluid appears to be in the pleural space. "B" is unlikely. Development of resistance should take more than 3 days, especially since this patient is on two drugs. "D" is unlikely given that this patient does not have a history of heart failure, is febrile, and has no JVD, etc. "E" is unlikely. None of the drugs that he is on is known to cause pleural effusions.

Question 4 of 7 On ultrasound, the effusion appears free flowing and not loculated. What is the most appropriate next step? A Referral for surgical drainage B Place a chest tube to drain the effusion C Perform an ultrasound-guided diagnostic thoracentesis at the bedside D Order two pizzas, one for you and one for the patient (you have both had a long day and are hungry)

The correct answer is "C." The patient has a relatively large pleural effusion. Ultrasound-guided thoracentesis is a good first step in evaluating this effusion. Ultrasound guidance is quickly becoming the standard of care as it has been shown to decrease complications of pneumothorax as well as the number of unsuccessful clinical attempts or "dry taps." Referral to a thoracic surgeon ("A") may eventually be necessary, but this would not be the first step. Placing a chest tube ("B") into an effusion is not recommended at this point, and the diagnostic study should be obtained first.

Question 5 of 6 You place a chest tube to drain the pleural effusion (free flowing on ultrasound) and continue the current antibiotic regimen. The patient does well and is discharged 1 week later on clarithromycin after sensitivities conclude that his organism is susceptible to it. Six weeks after the onset of illness, he returns for follow-up to ensure clearing of the chest x-ray. He is feeling well. He is alert and oriented, and his lung examination is now normal. There is no lymphadenopathy in the neck or supraclavicular areas. The x-ray still shows left lower lobe infiltrate, unchanged in size from the initial x-ray. The pleural effusion has resolved. Which of the following is the most appropriate next step in the evaluation and management of this patient? A Chest CT B Chest x-ray in 2 weeks C Chest x-ray in 6 weeks D Prescribe amoxicillin/clavulanate E Refer for bronchoscopy

The correct answer is "C." There are no clear guidelines regarding follow-up chest x-ray in patients who had pneumonia. The British Thoracic Society published recommendations in 2007 where it advised repeating chest x-ray in 6 weeks in patients with a smoking history. The reason for that recommendation is a chance that the infiltrate would obscure an underlying malignancy. Those over age 50 should also have follow-up x-ray. Bacteremic pneumococcal pneumonia has been associated with very slowly clearing x-rays, up to 3 to 5 months in some cases. Thus, repeating the chest x-ray in 2 weeks is unlikely to show resolution, so "B" is incorrect. In elderly patients, the chest x-ray takes longer to normalize than in younger patients. The patient is clinically doing well and does not require treatment for a persistent pulmonary infection, so "D" is incorrect. Chest CT ("A") and bronchoscopy ("E") would give more information, but in the absence of systemic symptoms, such as weight loss, persistent cough, hemoptysis, or fever, they are not indicated. It is important to consider the fact that this "infiltrate" may represent a neoplastic process if it does not resolve within several months.

Question 4 of 8 After several hours of noninvasive positive pressure ventilation, your patient is doing well, and he is transferred out of the intensive care unit (ICU). When you see him the next day his medications include inhaled bronchodilators, ceftriaxone, azithromycin, and prednisone. A chest x-ray shows no evidence of infiltrate. He is weaned from the positive pressure support to oxygen by nasal cannula and then, 2 days later, to room air. At discharge, he appears comfortable, with a respiratory rate of 14 and an oxygen saturation of 92% on room air. As you write his home-going prescriptions, what would be the best ongoing treatment for him? A Inhaled long-acting beta-agonist (LABA) + albuterol B Inhaled steroid + albuterol C Inhaled steroid + LABA + albuterol as needed D Albuterol E Albuterol and theophylline

The correct answer is "C." This is a bit tricky, but due to the severity of his exacerbation, "C" is the best choice. Compared to patients with asthma, where long-acting bronchodilators are not indicated as first-line therapy, COPD patients have a clear symptomatic benefit from long-acting bronchodilators. The two available choices are anticholinergics (LAMAs, or long-acting muscarinic agents [e.g., tiotropium, umeclidinium] or LABAs [e.g., salmeterol or formoterol]). Either choice is fine, and some prefer a LAMA as first line. An inhaled steroid does not modify the long-term decline in FEV1 in patients with COPD. However, it will reduce the frequency of exacerbations and thus improve health status. Current guidelines recommend addition of inhaled steroids with FEV1< 50% predicted and frequent exacerbations (e.g., 3 exacerbations in the last 3 years). See Table 3-3 for more on guideline-directed treatment. It is important to recognize there are adverse effects of inhaled corticosteroids (ICS) therapy including dysphonia, skin bruising, and oral candidiasis. In addition, ICS therapy may increase the incidence of pneumonia and cataracts, as well as diminish bone density. Therefore, for patients with COPD, other therapies are recommended (bronchodilators, smoking cessation, pulmonary rehabilitation), if possible, prior to ICS therapy. For patients who have persistent symptoms, repeated exacerbations, or severe exacerbations despite optimal long-acting bronchodilator regiment, ICS therapy is recommended.

Question 3 of 4 Which of the following is NOT a radiographic finding of granulomatosis with polyangiitis? A Nodules that may be cavitary B Alveolar opacification C Pleural opacities D Widened mediastinum

The correct answer is "D." A widened mediastinum is not one of the classic findings in GPA. However, one may, on occasion, see hilar adenopathy. All of the other choices can be found in GPA. In the patient's x-ray (Fig. 3-5), a right upper lobe mass is easily distinguished. In a young, non-smoking female presenting with these symptoms, such a lung mass should lead to the consideration of GPA or possibly an infectious process. She is less likely to have a malignant process.

Question 3 of 4 Your patient's 17-year-old brother is in the next examination room. He is a Boy Scout who just returned from a backpacking trip in the four corners area of New Mexico, Arizona, Colorado, and Utah. He has noted myalgias, fever, and chills. He knows that you are reviewing pulmonary medicine for your upcoming Board Examination, so he presents to your office complaining of dyspnea that has been getting markedly worse over the past several days. He has no URI symptoms such as coryza, rhinorrhea, ear pain, etc. Additionally, he has nausea, vomiting, and diarrhea with severe abdominal pain. His respiratory rate is 40 with an oxygen saturation of 88% on room air. ("See," he says, "I told you I'm sick.") You place him on nasal oxygen and order a chest x-ray, which shows bilateral pulmonary edema. Based on the epidemiology and chest x-ray appearance, your best guess at this point in the disease is: A Plague B Coccidiodomycosis C Hantavirus from Sin Nombre (No Name) strain D Noncardiogenic pulmonary edema from smoking paraquat (you never know what is going on at those Boy Scout camps) E Ischemic cardiomyopathy from cocaine use (you never know what is going on at those Boy Scout camps)

The correct answer is "C." This is a typical history and physical examination for hantavirus. The absence of URI symptoms, the presence of GI symptoms, and the non-cardiogenic pulmonary edema are all symptoms/signs of hantavirus. In fact, it may present as an acute abdomen. It is spread by aerosolization of mouse excrement or urine. "B" is incorrect. Although coccidiodomycosis, "Valley Fever," is found in the same geographical region, it presents with lower respiratory symptoms, a thin walled cavitary lesion, erythema nodosum (10%), and eosinophilia. Coccidiomycosis is generally a low-grade, subacute process that lasts weeks to months. It does not cause non-cardiogenic pulmonary edema. "A," pneumonic plague, while also found in the same area, causes high fever, bloody sputum, pleuritic chest pain, and develops over hours to days. It can be rapidly fatal if not recognized and treated within the first day. "D" and "E" are wrong for so many reasons that we won't bother to enumerate them here, but you never do know what goes on at camp.

Question 3 of 8 Your patient is now on 5 liters per minute (LPM) of oxygen via nasal cannula. You obtain an ABG: pH 7.29, PCO2 74 mm Hg, PO2 58 mm Hg. The patient is awake and alert but says that he still feels "like dirt." He remains tachypneic, in obvious respiratory distress, with a respiratory rate of 28. Albuterol and ipratropium are given via nebulizer. What is the best next step? A Clearly, he is failing therapy—emergently intubate B Increase his O2 to 100% via face mask C Initiate noninvasive positive pressure ventilation (e.g., BiPAP) D Start IV antibiotics E Obtain a chest CT with PE protocol

The correct answer is "C." This patient is retaining CO2 despite tachypnea and is in impending respiratory failure. He is also not oxygenating well despite low-flow oxygen. Noninvasive positive pressure support (BIPAP) can relieve hypercapnia and improve oxygenation by decreasing work of breathing without requiring intubation and its associated morbidity. Often IV antibiotics ("D") are used for empiric therapy in severe exacerbations of COPD, but again, improving the respiratory status comes first. He is hypoxic, but his main problem is CO2 retention—increasing his O2 delivery will not alleviate that (thus, "B" is incorrect). Although his respiratory status is tenuous, he is not in imminent respiratory failure, and intubation ("A") is not warranted at this time. Chest CT ("E") can rule out a PE in this hypoxic patient and may confirm COPD changes including emphysema, but will greatly delay treatment and would not change your immediate course of action—in other words, you would be barking up the wrong bronchial tree.

Question 6 of 8 Later that year, the same patient gets admitted to the hospital for community-acquired pneumonia. During his stay in the hospital, the hospitalist orders a CT chest "to rule out other things." The patient recovers from his infection and returns to you with the CD of his CT chest images. The reading of the CT chest describes a 2-cm pulmonary nodule in the right upper lobe along with extensive subcarinal lymphadenopathy. What is the next best step in management of this patient? A Repeat CT chest in 3 months B Repeat CT chest in 6 months C Refer for bronchoscopy with endobronchial ultrasound-guided biopsy D Refer to an oncologist E Refer for a mediastinoscopy

The correct answer is "C." This patient probably has a malignant disease and tissue is needed to either diagnose it or rule it out (see more later in this chapter). Bronchoscopy with endobronchial ultrasound (EBUS) guided fine-needle biopsies is a minimally invasive procedure that can relatively safely obtain a tissue sample for the pathologist. There is good evidence that EBUS has high sensitivity and specificity compared with PET scanning. "A" and "B" are incorrect as this patient has a nodule that is larger than 1 cm along with mediastinal lymphadenopathy. This needs to be worked up and cannot wait 3 to 6 months. "D" is incorrect. Your patient will likely need to see an oncologist, but you need to first provide a tissue diagnosis. "E," mediastinoscopy, would likely provide you with the diagnosis, but it is a far more invasive procedure than bronchoscopy and carries higher mortality and morbidity.

Question 1 of 7 A 57-year-old male with no prior medical history comes in to clinic with a 1-week history of right rib pain and low-back pain. The rib pain is worse with deep breaths and especially bothers him at night. There has been no trauma. He has lost 20 lb in the last 3 months. He has a cough productive of white sputum. He denies any other symptoms. He smokes one to two packs of cigarettes per day but does not drink alcohol. Vital signs: temperature 36.5°C, pulse rate 95 bpm, blood pressure 110/70 mm Hg, respiratory rate 16. On room air, his oxygen saturation is 96%. There is no adenopathy. His lung sounds are clear on the left and decreased on the right. There is dullness to percussion and decreased tactile fremitus over the right lower lung field. Based on this patient's history and physical examination, what do you expect to find on chest x-ray? A Normal chest x-ray B Cavitary lung lesion C Pleural effusion D Expanded lung fields E Pneumothorax

The correct answer is "C." This patient's findings suggest pleural effusion. Everything is diminished in pleural effusion: there is dullness to percussion, decreased breath sounds, decreased tactile fremitus, and decreased voice transmission. A cavitary lung lesion presents with either a normal examination or findings similar to an infiltrate (e.g., crackles, increased fremitus, and dullness to percussion). Expanded lung fields on chest x-ray are often seen in patients with COPD or asthma, and examination findings include prolonged expiratory phase, wheezing, and resonance to percussion. Pneumothorax presents with hyperresonance to percussion, decreased breath sounds, and decreased fremitus.

Question 1 of 3 A 42-year-old female comes to your office with a history of asthma that has been difficult to control. She relates symptoms that have been worsening over the last 4 to 6 weeks. She received two courses of oral corticosteroids during that time. Her symptoms improved with this therapy but quickly returned after completing the steroids. She denies fever, chills, and night sweats, but complains of a chronic cough productive of brownish-colored sputum. She is a homemaker in a suburban area and has no pets. Physical examination reveals wheezing throughout all lung fields but is otherwise normal. Laboratory evaluation includes CBC with increased eosinophils, normal C-reactive protein, and an elevated IgE level of 1250 ng/mL. A high-resolution CT scan of the chest reveals central bronchiectasis. What is the most likely diagnosis? A Hypersensitivity pneumonitis B Acute eosinophilic pneumonia C Allergic bronchopulmonary aspergillosis (ABPA) D Bacterial pneumonia E Churg-Strauss vasculitis

The correct answer is "C." This patient's history points to the diagnosis of ABPA, which is characterized by the presence of severe asthma, brownish mucus plugs, peripheral eosinophilia above 10%, elevated serum IgE, and central bronchiectasis. IgE elevation is required to be greater than 1,000 ng/mL. "A" is unlikely but a bit tricky. First, there is no history of exposure to a causative agent. Second, let's focus on symptoms. Constitutional symptoms—often fever—are present in the acute form of hypersensitivity pneumonitis. However, they need not be present in the subacute and chronic forms of the disease. So, based on symptoms, this could be hypersensitivity pneumonitis. However, the radiologic findings of hypersensitivity pneumonitis would include interstitial lung disease, rather than central bronchiectasis. Thus, this is not likely hypersensitivity pneumonitis. "B" and "D" are incorrect. Note that she has no significant constitutional symptoms that might be more typical of acute eosinophilic pneumonia or bacterial infection. You would also expect an infiltrate on the chest CT. "E" is incorrect. Churg-Strauss vasculitis is characterized by transient patchy interstitial infiltrates, fever, weight loss, elevated sedimentation rate, abnormal liver enzymes, and a peripheral blood eosinophilia >1,000 cells/μL. This is often related to using an oral steroid and a leukotriene inhibitor simultaneously. It is not related to inhaled steroid use. Extrapulmonary manifestations (CHF, pericarditis, tender subcutaneous nodules, peripheral neuropathy—most commonly mononeuritis multiplex) distinguish Churg-Strauss from other eosinophilic pulmonary conditions.

Question 3 of 4 You would advise this patient to: A Get a new job B Apply for disability C Use a respirator at work and avoid exposure to this toxin if possible D Sue the employer E Take up worm farming or monoculture in rhubarb

The correct answer is "C." Wearing an appropriate respirator at work can be beneficial. Avoiding exposure is even better. As for the other answers, you are a doctor not a lawyer or career counselor. Stick with what you know!

Question 3 of 6 All of the following are useful to help assess the risk of cancer in a patient with a solitary pulmonary nodule (SPN) EXCEPT: A Smoking status B Age C Diameter of the nodule D Gender

The correct answer is "D." Determining the probability of cancer in patients with an SPN is an inexact science. The risk of cancer is generally assessed as low, intermediate, or high based on patient and radiograph characteristics. Although men are slightly overrepresented in lung cancer diagnoses, this is generally thought to be due to greater smoking rates in men and to occupational hazards. Gender itself does not help to risk-stratify patients with an SPN. Smoking increases the risk of an SPN being cancer, with greater use increasing the risk of cancer. As with most cancers, increasing age is associated with a higher risk. The diameter of the nodule is also important. If the diameter is <8 mm, the risk of cancer is low. When the diameter is ≥3 cm, the SPN is now referred to as a "pulmonary mass" and is highly likely to be cancerous. An SPN >3 cm in diameter should be considered cancer until proven otherwise. The Fleischner Society has published an updated set of widely accepted recommendations regarding follow-up of incidentally found pulmonary nodules in 2017. The guideline does not apply to patients <35 years or with a history of cancer or immunosuppression (see Table 3-10).

Question 7 of 8 Your patient returns with increasing dyspnea now at rest. His biopsy was negative. Despite both of you being blue in the face, he has not quit smoking. Criteria for the use of continuous low-flow oxygen in those with COPD include all of the following EXCEPT: A PO2 <55 mm Hg B Oxygen saturation <88% C PO2 of <59 mm Hg with evidence of cor pulmonale D Episodic sleep apnea-related desaturations at night

The correct answer is "D." Episodic sleep apnea-related oxygen desaturations, while a cause for concern and amenable to treatment (e.g., CPAP), are not one of the criteria for the use of continuous low-flow oxygen. The other choices are correct. "C" deserves some special attention. Evidence of cor pulmonale can include "p-pulmonale" on ECG, peripheral edema, or a hematocrit >55%.

Question 1 of 6 A 74-year-old male presents to your ED for weakness, cough, and fatigue. His wife relates an incomplete recovery since his myocardial infarction last year. He continues to have poor appetite and listlessness, and she thinks that he may be depressed. He is short of breath and confused. His wife says that yesterday he developed a fever, chills, and a new cough productive of white sputum. His past medical history is otherwise remarkable for a cholecystectomy. He is taking aspirin (secondary prevention, not primary!), metoprolol, and atorvastatin. Vitals: temperature 39°C, respiratory rate 30, pulse 90 bpm, blood pressure 140/80 mm Hg. Oxygen saturation on room air is 90%. He is thin, pale, and oriented to person only. The lung examination is remarkable for rales in the left lower field, with dullness to percussion and increased tactile fremitus. The remainder of the examination is normal. The chest x-ray shows a left lower lobe infiltrate. Other laboratory data currently available: hemoglobin 12.4 g/dL, WBC 14,100/mm3, platelets 340,000/mm3, creatinine 1.9 mg/dL, BUN 50 mg/dL, and normal electrolytes, troponin, and CK. An ECG shows normal sinus rhythm. What is your next step in managing this patient's medical condition? A Place a chest tube on the left B Perform chest CT C Administer inhaled bronchodilators D Administer parenteral antibiotics E Perform intubation and mechanical ventilation

The correct answer is "D." Given the clinical picture and chest x-ray findings, the patient most likely has community-acquired pneumonia. Therefore, the administration of parenteral antibiotics is the best choice. "A" is incorrect. Since there is no effusion, a chest tube would be useless. "B" is incorrect. CT is not required in this straightforward case of pneumonia. "C" is incorrect. The patient is not wheezing and there is no indication for bronchodilators at this time. As for "E," since your patient's respiratory status is stable, he does not require intubation.

Question 4 of 4 Laboratory findings suggestive of hantavirus include all of the following EXCEPT: A Thrombocytopenia B Leukocytosis with a left shift C A lymphocytic predominance D An immunoblast count <10%

The correct answer is "D." In fact, thrombocytopenia, a 10% or greater immunoblast count, and a left shift constitute the so-called diagnostic triad in a patient with appropriate clinical findings. Immunoblasts are the most immature cell in the lymphocyte line (they still enjoy scribbling on walls and drinking irresponsibly). Overall, case fatality rate of hantavirus is up to 50%. Care is supportive. Extra-corporeal membrane oxygenation may be used in seriously ill patients. There is often an oliguric phase that needs careful management to prevent fluid overload. This can be problematic because patients are often hypotensive and there is a proclivity toward giving them fluids.

Question 7 of 9 Your patient does quite well over the next year, having very few exacerbations. During one of her visits, you note slightly edematous nasal mucosa and nasal polyps. You prescribe intranasal steroids. Then, one night when you are on call, she comes in severely dyspneic with audible wheezing. She talks in two- or three-word phrases. She reports a headache today, which she treated with aspirin (something she never takes, but a friend gave it to her thinking it was acetaminophen). Her asthma attack started about an hour after the aspirin dose. She has been otherwise well. She denies fever, rhinorrhea, nasal congestion, and sore throat. Her respiratory rate is 40, heart rate 120 bpm, and oxygen saturation 88% on room air. She has poor air movement on auscultation of her lung fields. Which of the following is the most likely reason for this patient's acute exacerbation of asthma? A Viral upper respiratory infection (URI) B Sinusitis C Noncompliance with inhaled albuterol D Sensitivity to aspirin E Noncompliance with nasal steroids

The correct answer is "D." It is likely that this patient has aspirin sensitivity. Up to 10% of adults with asthma have the clinical triad of asthma, aspirin sensitivity, and nasal polyposis. Patients with asthma should be warned about the potential for exacerbations resulting from consumption of aspirin and nonsteroidal anti-inflammatory drugs (NSAIDs). The drug-induced bronchial constriction caused by these medications can have an abrupt onset with severe symptoms. Patients with aspirin sensitivity can be desensitized with daily administration of small amounts of aspirin, but this should be done carefully with close supervision. Although viral URIs frequently cause exacerbations of asthma, your patient did not report antecedent symptoms of such an infection. Further discussion of the treatment of an acute asthma exacerbation can be found in Chapter 1, "Emergency Medicine."

Question 2 of 3 Which of the following would be the next best step in confirming the diagnosis? A Sputum cultures B Transbronchial biopsy C Methacholine challenge D Allergy skin testing for Aspergillus species E p-ANCA

The correct answer is "D." Most but not all of the following criteria (Table 3-11) need to be present in order to make the diagnosis of ABPA. Transbronchial biopsy is unnecessarily invasive, and the other tests will not help to confirm the diagnosis.

Question 8 of 8 Concerning hypoxemic patients with COPD (resting oxygen saturation of <88%), which of the following is true? A Patients on continuous, low-flow O2 become oxygen dependent and cannot function without it B Continuous low-flow O2 used for at least 8 hours a day helps to reverse pulmonary hypertension C Concurrent smoking is a contraindication to the prescribing continuous low-flow home O2 because patients are spontaneously combusting right and left (like Spinal Tap drummers) D Low-flow O2 used at least 15 hours a day significantly enhances survival E Low-flow O2 is a well-tolerated and effective treatment for obstructive sleep apnea

The correct answer is "D." Patients who use continuous low-flow O2 at home have an improved rate of survival. Patients should be encouraged to use O2 at least 15 hours a day, if possible, to obtain this benefit. "B" is incorrect because patients need at least 15 hours of O2 per day to have any significant benefit with regard to pulmonary hypertension. "C" is incorrect. Clearly, smoking while on O2 is not a good idea, but patients can turn off their O2 supply while smoking.

Question 5 of 7 Ultrasound-guided thoracentesis is successful in obtaining fluid. The fluid is amber and cloudy, with a pH 7.3, lactate dehydrogenase (LDH) 800 IU/L, glucose 65 mg/dL, total protein 5.5 g/dL, WBC 1,300/mm3, RBC 50,000/mm3. Serum studies done the same day include LDH 155 IU/L, glucose 99 mg/dL, and total protein 7 g/dL. Cytology, Gram stain, and culture of the pleural fluid are pending. Which of the following is the most accurate statement regarding the pleural fluid analysis? A The fluid is due to infection B The fluid is due to cancer C The fluid is a transudate D The fluid is an exudate

The correct answer is "D." Pleural effusions are broadly categorized as exudates and transudates (see Tables 3-8 and 3-9). Such a categorization helps to narrow the differential diagnosis. In this case, several elements of the pleural fluid are consistent with an exudate. LDH and protein can be used to determine whether the pleural fluid is transudative or exudative. Per Light's criteria, a pleural effusion is suggestive of an exudative process if two of any of the following criteria are met: pleural fluid LDH >2/3 the upper limit of normal serum LDH, a pleural LDH:serum LDH ratio of >0.6, and a pleural protein:serum protein ratio of >0.5. All three of these indicators point to an exudate in this case. Also, exudative effusions tend to have a higher degree of cellularity than transudative effusions. With the information given, it is difficult to determine if the effusion is related to infection, cancer, or some other process.

Question 2 of 7 Your suspicions are confirmed. The chest x-ray shows obliteration of the right hemidiaphragm, and the posterior costophrenic angle is obscured on the lateral view, consistent with pleural effusion. There is also a right upper lobe lung mass. Which of the following will provide the most information and guidance for your thoracentesis? A Supine chest x-ray B Chest CT C Lateral decubitus chest x-ray D Chest ultrasound E Apical view chest radiograph

The correct answer is "D." Prior to performing a thoracentesis, you must know whether the effusion is loculated or freely flowing. Portable ultrasound has become a validated and widely accepted modality to diagnose and assess pleural effusion. Also, ultrasonography has been found to be more sensitive for detection of pleural fluid than a chest radiograph. Chest CT is somewhat more sensitive but more cumbersome, exposing the patient to additional radiation, and does not allow a bedside diagnosis and treatment. A decubitus film, with the affected side down, would allow you to see the effusion "layer out" unless it is loculated but again is less sensitive. A supine chest x-ray may cause the effusion to "layer out" too, but you will not be able to see it as well, which is why effusions may be missed when a patient is unable to stand or sit upright for his x-ray.

Question 2 of 5 Which of the following is NOT commonly associated with sarcoidosis? A Hypercalcemia B Elevated ACE levels C Reduced diffusion capacity D Hypothyroidism E Facial or peripheral nerve palsy

The correct answer is "D." Sarcoidosis is marked by the presence of non-caseating granulomas. While sarcoid can infiltrate the thyroid, it rarely, if ever, causes hypothyroidism. Pulmonary sarcoidosis includes a decreased diffusion capacity and decreased vital capacity. Other laboratory findings include hypercalcemia, hypercalciuria, elevated liver and pancreatic enzymes, and elevated ACE levels. Neurologic involvement occurs in up to 5% of patients and frequently presents as facial paralysis but may present as any central nervous system lesion. Peripheral nerves may also be involved.

Question 3 of 6 On the basis of your assessment of his risk, you decide to admit this patient to the hospital. An IV is in place. Which of the following IV antibiotic regimens do you choose? A Penicillin B Azithromycin C Penicillin and gentamicin D Azithromycin and ceftriaxone E Piperacillin/tazobactam and ciprofloxacin

The correct answer is "D." The 2007 Infectious Disease Society of America /American Thoracic Society (IDSA/ATS) guideline for community-acquired pneumonia (still not updated as of the writing of this book) recommends that for community-acquired pneumonia treated in the hospital setting, the optimal antibiotic regimen must offer good coverage of S. pneumoniae, H. influenzae, and atypical organisms such as Mycoplasma and Chlamydia species. Most S. pneumoniae bacteria are resistant to penicillin ("A") and about 20% to 30% are resistant to macrolides such as azithromycin ("B"). Therefore, these agents should not be used alone in the treatment of pneumonia in hospitalized patients. Gentamicin ("C") has no activity against S. pneumoniae but has a role in P. aeruginosa infections. Ceftriaxone offers good Gram-negative coverage and activity against S. pneumoniae. Azithromycin covers atypical organisms. For these reasons, "D" is the best choice. An alternative regimen would be monotherapy with a respiratory fluoroquinolone, such as moxifloxacin or levofloxacin (remembering the cautions associated with fluoroquinolones). The combination of piperacillin/tazobactam with ciprofloxacin ("E") is reserved for patients with more severe pneumonia, requiring ventilation and ICU care.

Question 2 of 4 What is the best next step in the diagnosis of this process? A Bronchoscopy B Sputum cultures C Blood cultures D Chest CT E Open-lung biopsy

The correct answer is "D." The chest x-ray demonstrates a cavitary lesion in the right upper lobe. Chest CT is warranted for further characterization of the lesion. From history, examination, and chest x-ray, it is not possible to determine whether the lesion is an abscess or a malignant process. An indolent course with low-grade fever is characteristic of lung abscess. However, the pre-existing squamous cell carcinoma has potential to have spread to the lungs, and squamous cell carcinoma is known to cause cavitations. Culture of sputum and blood, including evaluation of first morning sputum for acid-fast bacilli (AFB), will be an essential part of the assessment but may not yield as much information as chest CT, and sputum culture should be done in conjunction with cytology and Gram stain. Bronchoscopy should be postponed until CT results are available. Bronchoscopic biopsy is potentially detrimental if the lesion is an abscess since the airway could flood with pus if the entire cavity wall is penetrated.

Question 7 of 9 As keenly suspected, Ms. Bellum's CTA of the chest reveals a moderate-sized pulmonary embolus in the left pulmonary artery. Her vital signs are still stable and her pain is well-controlled with oral hydrocodone. She is surprised by the diagnosis you give her but appears to be taking it in stride. What is the optimal management plan for the patient moving forward? A Bolus her with unfractionated heparin (UFH), start her on oral warfarin, and discharge her to home B Start the patient on low-molecular-weight heparin (LMWH), initiate oral warfarin therapy, and admit the patient to the family medicine service C Start apixaban (Eliquis) or rivaroxaban (Xaralto) and discharge to home D Start her on oral warfarin and discharge her to home with primary care follow-up in the next 2 to 3 days E B or C

The correct answer is "E." Let's dissect why. "A" and "D" are both incorrect because they do not include adequate anticoagulation. Certainly "B" is the classic approach. However, this patient meets the "HESTIA" criteria for early discharge from the ED (see Table 3-5), and need not be admitted, so many physicians now would choose discharge on a direct anticoagulant. Early discharge has been found to be safe in low-risk patients [those with all HESTIA criteria negative and a low Pulmonary Embolism Severity Index (Ann Intern Med. 2018;169(12):855-865 and Acad Emerg Med. 2018 Sep; 25(9):997-1003]. To calculate the PESI, use an online calculator, such as https://www.mdcalc.com/pulmonary-embolism-severity-index-pesi. With regard to selecting an anticoagulant, current evidence does not support the use of one agent over another; UFH, LMWH, and fondaparinux followed by warfarin are all appropriate as are the newer agents (e.g., apixaban, dabigatran, and rivaroxaban). Both warfarin and dabigatran require an overlap with heparin. With UFH, LMWH, or fondaparinux transitioning to warfarin, continue these drugs until the patient's INR has been therapeutic (INR of 2-3) for at least 24 hours and the overlap is for at least 5 days. Dabigatran requires 5 days of heparin overlap (and of course we don't check the INR). Avoid edoxaban (Savaysa) if possible. It can only be used in those with a CrCl of less than 95 mL/min. Also, it requires dose adjustment for those with a CrCl of between 50 and 15 mL/min (do not use under CrCl 15 mL/min).

Question 6 of 7 The pleural fluid cytology comes back negative. The patient's symptoms and examination have not changed. Repeat radiograph still shows an upper lobe mass. What is the most appropriate next step in approaching this pleural effusion? A Await pleural fluid culture results B Perform bedside chest tube drainage of the effusion C Refer for surgical evacuation of the effusion D Refer for bronchoscopy E Place a chest tube for chemical pleurodesis

The correct answer is "D." The effusion is clearly exudative, and the patient appears to have a lung mass. Biopsy of the lung mass via bronchoscopy is indicated. A negative pleural fluid cytology does not rule out lung cancer. Positive cytology indicates advanced stage lung cancer. Chest tube drainage of a pleural effusion is not recommended except under extraordinary circumstances. Intermittent thoracentesis is preferred and has lower morbidity. Surgical evacuation of the fluid would be indicated if the patient were symptomatic, and the effusion was loculated and/or related to infection. If the effusion grows, or is drained and recurs, it may respond to pleurodesis. Otherwise, pleurodesis is not indicated at this time. 57% of users answered correctly.

Question 2 of 9 After further verbal probing, you discover that Ms. Bellum recently completed her menstrual cycle. The other presented questions turned up no risk factors. However, your smooth segue did reveal that she takes low-dose estrogen for birth control. You also learn that her aunt had a blood clot in her leg once. She has no further details but does not think that her aunt had any further complications from this condition. Regardless, PE just took a violent leap to the top of your differential. You glance at her vitals (temperature 37.1°C, heart rate 92 bpm, blood pressure 129/68 mm Hg, respiratory rate 21, SpO2 95% on room air) and notice that she appears mildly uncomfortable but not in any acute distress. Her physical examination is entirely unremarkable. You order an ECG to evaluate for potential cardiac etiologies for her symptoms. Assuming Ms. Bellum does have a PE, what is her ECG most likely to show? A S1Q3T3 B Nonspecific ST-T wave changes C Sinus tachycardia D Normal sinus rhythm E Multifocal atrial tachycardia

The correct answer is "D." The most common ECG finding associated with the diagnosis of PE remains normal sinus rhythm. With that said, the most common arrhythmia found in patients with a PE is sinus tachycardia. But alas, Sarah had a normal heart rate. The other choices can certainly be found with this condition but are far less frequent. Of note, the "textbook" S1Q3T3 ECG rarely occurs and historically traces back to a handful of patients in the 1930s that had massive pulmonary emboli. Even if you do spot this pattern on an ECG, it is not specific enough to confirm the diagnosis. In the end, the clinical signs attributed to pulmonary emboli (such as the shortness of breath and chest pain) are more valuable than any abnormal ECG finding.

Question 1 of 4 While you are covering the ED, a 60-year-old female comes in by ambulance. She is unresponsive, and her husband states that he found her 30 minutes ago surrounded by bottles of pills and an empty bottle of vodka. She has a history of COPD, hypertension, osteoarthritis, and depression. The EMTs brought in her pill bottles, which include lorazepam, acetaminophen/hydrocodone, hydrochlorothiazide, aspirin, and sertraline. Only a few tablets are left in the bottle of hydrochlorothiazide. She is wearing a nonrebreather facemask with 50% oxygen. Her respirations are shallow with a rate of 8. The remainder of her vitals: temperature 36°C, blood pressure 90/50 mm Hg, and pulse rate 90 bpm. Oxygen saturation is 88% and increases to 94% with some assisted breaths. One nurse is obtaining a blood gas while another gives naloxone. You decide that this patient cannot protect her airway and choose to intubate her. The blood gas drawn just before intubation shows pH 7.16, PaCO2 60 mm Hg, and PaO2 40 mm Hg. These findings imply which of the following processes? A Metabolic acidosis B Metabolic alkalosis C Respiratory alkalosis D Mixed metabolic/respiratory acidosis E Mixed metabolic/respiratory alkalosis

The correct answer is "D." The pH is acidotic (<7.4). In a patient whose baseline PaCO2 is not known to you, you might assume her PaCO2 is usually 40 mm Hg, which is the accepted normal for most patients. If the acidosis is purely due to acute respiratory changes and CO2 retention, a rise in PaCO2 of 10 should be accompanied by a fall in pH equal to 0.08. In this case, the change in PaCO2 is 20. So, 20/10 × 0.08 = 2 × 0.08 = 0.16 resulting in a pH of 7.4 − 0.16 = 7.24. However, this patient's pH is measured at 7.16, lower than expected for a pure respiratory acidosis presenting acutely. Thus, you can determine that the acidosis is both metabolic (perhaps from lactic acidosis from hypoperfusion) and respiratory.

Question 3 of 4 He does not respond after 2 months of empiric treatment, and he is becoming more concerned. The examination is unchanged. Spirometry is normal with a normal flow volume loop. Which of the following management options is LEAST likely to benefit this patient? A Combination antihistamine and decongestant B Inhaled corticosteroid C Inhaled beta-2 agonist D Antibiotics

The correct answer is "D." This patient has no signs or symptoms of sinusitis or bacterial pulmonary infection, so treating with an antibiotic is inappropriate and unlikely to help. However, some form of empiric therapy might be tried. He could have postnasal drainage without signs on physical examination, and empiric therapy with combination antihistamine and decongestant may improve the cough. Inhaled corticosteroids and beta-2 agonists are the mainstay of chronic asthma therapy and may help relieve this patient's chronic cough. This patient could yet have "cough-variant asthma" despite normal spirometry results.

Question 6 of 6 In one of life's funny little coincidences, the next day you diagnose this patient's 36-year-old healthy son with a community-acquired pneumonia. He has a fever, cough, and left lower lobe infiltrate on chest x-ray, but he is hemodynamically stable. You determine that he's fit for outpatient management. Which of the following drug regimens is appropriate for the treatment of this patient in the outpatient setting? A Cephalexin 250 to 500 mg PO QID for 10 days B Penicillin V 250 mg TID for 10 days C Clarithromycin 500 mg BID for 10 days D Doxycycline 100 mg BID for 10 days E C or D

The correct answer is "E." The treatment of community-acquired pneumonia requires coverage of "typical" and "atypical" organisms. Neither "A" nor "B" covers atypical organisms. Guideline-recommended choices for the outpatient treatment of community-acquired pneumonia include doxycycline and macrolides such as clarithromycin (doxycycline has the least resistance). Additional options include the respiratory fluoroquinolones such as moxifloxacin, gemifloxacin, or levofloxacin. However, respiratory fluoroquinolones should not be used in all cases. The IDSA/ATS consensus guidelines recommend that respiratory fluoroquinolones be reserved for patients with serious underlying disease (e.g., COPD, diabetes, immunocompromised states). Of the appropriate regimens, doxycycline and erythromycin are the least expensive, but erythromycin is associated with a high rate of gastrointestinal intolerance.

Question 2 of 4 You obtain the chest x-ray pictured in Fig. 3-5. You obtain the following laboratory results: CBC: Leukocytosis, thrombocytosis, and normochromic, normocytic anemia ESR: 70 mm/hr Urine dipstick: Positive for protein, heme, and red cells FIGURE 3-5. Patient's chest x-ray. View Full Size|Favorite Figure |Download Slide (.ppt) Which of the following tests will best assist you in the diagnosis of this patient? A Antineutrophil cytoplasmic antibody (ANCA) B Antiglomerular basement membrane antibody C Antinuclear antibody (ANA) D A and B E A and C

The correct answer is "D." This patient is presenting with the classic triad of granulomatosis with polyangiitis (GPA), a disease of the upper respiratory tract, lower respiratory tract, and kidneys, formerly known as Wegener's granulomatosis (as a response to Wegener's association with the Nazi Party, professional bodies and journals have replaced his name with a descriptive name). She has some of the additional signs and symptoms associated with GPA, as well. Common findings include pleuritic chest pain, myalgias, arthralgias, ptosis, fever, weight loss, and purpuric skin lesions, among others. ANCA, and especially c-ANCA that is more specific for GPA, is present in up to 90% of patients with GPA. An ANA is not helpful in diagnosing GPA. An antiglomerular basement membrane antibody (anti-GBM) may be helpful in diagnosing Goodpasture syndrome (named for an American pathologist—not a Nazi—so he gets to keep the eponym). Goodpasture syndrome can be clinically easily confused with GPA; they both present with respiratory and renal involvement. Thus, anti-GBM antibody will be helpful in differentiating these two. However, about 10% of patients with Goodpasture syndrome will also have GPA—just to add to the confusion. For a partial list of causes of hemoptysis, see Table 3-7.

Question 2 of 6 Which of the following is NOT considered a benign pattern of calcification on CT scan? A Diffuse, homogeneous calcification B Central calcification C Laminar calcification D Spiculated, irregular calcification E "Popcorn" calcification

The correct answer is "D." We are accustomed to thinking of calcified nodules as being benign, but that is not always the case. Irregular, spiculated calcification is not reassuring. The other options are considered indicative of a benign lesion. Two patterns on CT are relatively specific for cancer: a scalloped border and the corona radiata sign, which is composed of fine linear strands extending out from the nodule.

Question 2 of 6 Based on patient-specific characteristics and your knowledge of causative factors involved in pneumonia, which of the following is LEAST likely to be the agent causing this patient's infection? A Mycoplasma pneumoniae B S. pneumoniae C Haemophilus influenzae D Pseudomonas aeruginosa

The correct answer is "D." When a pathogen is identified in adult community-acquired pneumonia, it is usually S. pneumonia ("B"). In fact, S. pneumoniae makes up 40% to 60% of all cases of community-acquired pneumonia in the elderly. Non-typeable H. influenzae ("C") composes about 5% to 10% of cases. Mycoplasma pneumoniae ("A") is implicated in 5%, and it is more common in young adults. P. aeruginosa pneumonia is uncommon in healthy elders and more likely to occur in patients with serious underlying lung disease or immunodeficiency (think cystic fibrosis). Approximately 5% of patients or more are infected with multiple agents.

Question 3 of 7 Relative and absolute contraindications to thoracentesis include all of the following EXCEPT: A Herpes zoster in the area of needle placement B Coagulopathy C Diaphragmatic rupture D Positive pressure ventilation E History of recurrent laryngeal nerve injury or compromise

The correct answer is "E." Absolute contraindications include chest wall compromise (e.g., burn, cellulitis, herpes zoster, ruptured diaphragm) and cases where chest tube thoracostomy would be more appropriate. Relative contraindications are poor patient cooperation, coagulopathy, anticoagulation therapy, very small effusions (<10 mm on decubitus film view), positive pressure ventilation, and pleural adhesions.

Question 5 of 8 Holding his inhalers and smiling toothlessly, your patient asks, "Which of these is going to keep me alive—just in case I can't afford them all?" Which of the following medication regimens has demonstrated decreased mortality in the treatment of stable COPD? A Inhaled tiotropium (LAMA) B Inhaled salmeterol (LABA) C Inhaled ipratropium D Inhaled corticosteroid E None of the above

The correct answer is "E." Aside from oxygen, no medical therapy has clearly demonstrated a mortality benefit for stable COPD. Again refer to Table 3-3 for stage of COPD and appropriate treatment. Another therapy to consider in advanced COPD is pulmonary rehabilitation (PR). While having no definite effect on survival, PR improves dyspnea and quality-of-life scores while reducing the number of hospitalizations and days in hospital. Also, there are oral phosphodiesterase inhibitors (e.g., roflumilast [Daliresp]) for severe COPD. As you probably already know, your authors are curmudgeons who don't like anything new. But roflumilast is particularly on our bad list. It increases FEV1 by 45 mL over placebo. It has no effect on quality of life and exacerbations are 1.3/year with placebo and 1.2/year with roflumilast. It also causes diarrhea and depression/suicidality. Nonetheless, you should know about it. Another therapy for COPD patients with recurrent exacerbations despite optimal therapy is chronic azithromycin, which may reduce the frequency of exacerbations (Int J Chron Obstruct Pulmon Dis. 2018;13:3813-3829). However, this should be avoided in patients with a long QT interval or if there are major concerns about hearing loss. This therapy needs to be carefully considered as studies have revealed that during 5 days of azithromycin therapy, there was a small absolute increase in cardiovascular deaths among patients with high baseline risk of cardiovascular disease (N Engl J Med. 2012;366:1881-1890).

Question 1 of 9 Ms. Sarah Bellum (if you're not smiling, try saying the name out loud or the joke is just lame ...) is a 32-year-old female who presents to your ED with shortness of breath. She just flew home from the International Conference on Balance and Coordination in London. Immediately after walking through her front door, she became acutely short of breath. This is associated with some moderately sharp chest pain located along the left side of her chest. The pain seems worse when she attempts to breathe deeply. Which important question(s) do you next ask Ms. Bellum? A Do you smoke cigarettes? B When was your last menstrual period? C Have you had surgery recently? D Do you have a history of kidney disease? E All of the above.

The correct answer is "E." Each of these questions addresses risk factors associated with pulmonary embolism (PE) and/or deep vein thrombosis (DVT). Smoking cigarettes and recent surgery are strong risk factors, as is an active pregnancy. Addressing a patient's menstrual cycle serves as a natural segue to a discussion about the use of oral contraceptives, which, too, is a prominent risk factor. As for renal disease, nephrotic syndrome has been associated with an increased risk of PE.

Question 2 of 2 In most adults with bronchiectasis, its cause is: A Genetic B Pseudomonas infection C Tobacco smoking D Allergic bronchopulmonary aspergillosis (ABPA) E Unknown

The correct answer is "E." There are limited data regarding the etiology of bronchiectasis, but many conditions and environmental exposures seem to have an association. In most patients, no cause is identified. Children are more likely than adults to have an identified etiology of their bronchiectasis, and the most common causes in kids are foreign body aspiration, cystic fibrosis, and gastroesophageal reflux. Identified etiologies in adults include those mentioned for children and pulmonary infections, ABPA, COPD, rheumatic or other inflammatory diseases, immunodeficiencies, and cigarette smoking.

Question 4 of 6 Later that week, your patient returns with his CT scan in hand. His cough is somewhat better (therapeutic CT scan radiation therapy ... you know, like those CT scans in California that were mis-programmed and cooking people's brains accidentally? Really ...). You review the CT scan with him. It shows a round, smooth nodule measuring 2 cm in diameter and located in the periphery of the right lower lobe. There are no calcifications in the nodule and no other abnormalities. Which of the following is the most appropriate next step? A Referral for bronchoscopy B High-resolution CT scan every 3 months C Chest x-ray every 3 months D Bone scan E Referral to a thoracic surgeon

The correct answer is "E." This patient needs a biopsy. There are several factors that put your patient at higher risk of having a malignant cause for the SPN, including his age and tobacco use. These put him into an intermediate- to high-risk category for cancer. Although the nodule is smooth on CT, its size is >8 mm and there are no calcifications. This patient should be referred for transthoracic fine-needle biopsy or open biopsy. "A" is tempting but incorrect. Bronchoscopy is insensitive in the peripheral lung, especially when the lesion is relatively small. "B" and "C" are also wrong here but are appropriate in other settings. In this case, repeat imaging over time may delay a diagnosis of malignancy. Without symptoms of bone pain or confirmation that the SPN is a cancer that might metastasize to bone, a bone scan will have a very low yield.


Ensembles d'études connexes

General Psychology Unit 2 (Chapters 5-8)

View Set

MKTG 361 EXAM 4 REVIEW QUESTIONS

View Set

Lesson 8: Money,Banking, Saving, Investing

View Set

Ch. 7: Radio, Recording, and Popular Music

View Set